Ελάχιστος ακέραιος

Συντονιστές: Demetres, socrates, silouan

socrates
Επιμελητής
Δημοσιεύσεις: 6461
Εγγραφή: Δευ Μαρ 09, 2009 1:47 pm
Τοποθεσία: Θεσσαλονίκη
Επικοινωνία:

Ελάχιστος ακέραιος

#1

Μη αναγνωσμένη δημοσίευση από socrates » Σάβ Φεβ 27, 2016 12:30 am

Έστω n θετικός ακέραιος και f(n) ο ελάχιστος ακέραιος τέτοιος ώστε σε οποιουσδήποτε f(n) θετικούς ακεραίους να μπορούμε να επιλέξουμε άρτιο πλήθος από αυτούς των οποίων των άθροισμα (των επιλεγέντων) να διαιρείται με το n. Βρείτε το f(n).


Θανάσης Κοντογεώργης
socrates
Επιμελητής
Δημοσιεύσεις: 6461
Εγγραφή: Δευ Μαρ 09, 2009 1:47 pm
Τοποθεσία: Θεσσαλονίκη
Επικοινωνία:

Re: Ελάχιστος ακέραιος

#2

Μη αναγνωσμένη δημοσίευση από socrates » Δευ Νοέμ 21, 2016 10:22 pm

Επαναφορά! :)


Θανάσης Κοντογεώργης
Άβαταρ μέλους
Demetres
Γενικός Συντονιστής
Δημοσιεύσεις: 8989
Εγγραφή: Δευ Ιαν 19, 2009 5:16 pm
Τοποθεσία: Λεμεσός/Πύλα
Επικοινωνία:

Re: Ελάχιστος ακέραιος

#3

Μη αναγνωσμένη δημοσίευση από Demetres » Δευ Νοέμ 28, 2016 4:44 pm

Την θεωρώ δύσκολη άσκηση γι' αυτό και θα την μεταφέρω από το «Επίπεδο Αρχιμήδη (Seniors)» στο «Προχωρημένο Επίπεδο (Seniors)»

Μπήκε πέρσι στον διαγωνισμό επιλογής του Ιράν και είναι η άσκηση 69 εδώ.

Βάζω την νύξη που είχα δώσει:
Η απάντηση είναι k = 2n αν n περιττός και k = n+1 αν n άρτιος. Οι κατασκευές που δείχνουν τα κάτω φράγματα είναι σχετικές απλές. Για τις αποδείξεις για τα άνω φράγματα, η πιο κάτω γνωστή πρόταση είναι αρκετά χρήσιμη:

Έστω φυσικός n και έστω ένα σύνολο S από n φυσικούς αριθμούς. Τότε υπάρχει μη κενό υποσύνολο του S με άθροισμα στοιχείων πολλαπλάσιο του n.

Η απόδειξη για το k \leqslant 2n είναι και αυτή απλή δεδομένου της πιο πάνω πρότασης. Η δυσκολία είναι στην απόδειξη k \leqslant n+1 για n άρτιο. Το πρώτο βήμα είναι ο χωρισμός του A σε δισύνολα με άρτιο άθροισμα το κάθε ένα καθώς και σε ένα μονοσύνολο το οποίο όμως δεν χρησιμοποιείται περαιτέρω.


dement
Διευθύνον Μέλος
Δημοσιεύσεις: 1417
Εγγραφή: Τρί Δεκ 23, 2008 10:11 am

Re: Ελάχιστος ακέραιος

#4

Μη αναγνωσμένη δημοσίευση από dement » Τρί Νοέμ 29, 2016 12:32 pm

Απέφυγα να δω την νύξη, ίσως υπάρχει πιο σύντομος τρόπος. Συμβολίζω με (a_k) τους επιλεγέντες αριθμούς.

Έστω n = 2k+1. Για πλήθος όρων 4k+1 μπορούμε να επιλέξουμε a_m \equiv 1 \mod 2k+1 για κάθε m (και το ζητούμενο είναι αδύνατο). Αντίθετα, για πλήθος 4k+2, μεταξύ των 4k+3 ισοτιμιών των \displaystyle \sum_{m=1}^t a_m \ (0 \leqslant t \leqslant 4k+2) θα υπάρχουν τρεις ίδιες (έστω p < q < r), οπότε ένα από τα \displaystyle \sum_{m=p+1}^q a_m, \sum_{m=q+1}^r a_m, \sum_{m=p+1}^r a_m, που διαιρούνται με το 2k+1, θα έχει άρτιο πλήθος όρων. Έτσι, f(2k+1) = 4k + 2.

Παρατηρούμε ότι χωρίς τον περιορισμό του αρτίου πλήθους θα ίσχυε f(n) = n για κάθε n (περιττό ή άρτιο). Πράγματι, για πλήθος n-1 μπορούμε να επιλέξουμε όλους τους αριθμούς ισότιμους με 1 \mod n (και το ζητούμενο είναι αδύνατο). Αντίθετα, για πλήθος n, μεταξύ των n+1 ισοτιμιών των \displaystyle \sum_{k=1}^m a_k \ (0 \leqslant m \leqslant n) θα υπάρχουν δύο ίδιες (έστω p < q), οπότε \displaystyle n \mid \sum_{k=p+1}^q a_k.

Για n = 2k, f(2k) > 2k αφού μπορούμε να επιλέξουμε a_1 \equiv 2 \mod 2k και a_m \equiv 1 \mod 2k για m > 1 (και το ζητούμενο είναι αδύνατο). Για πλήθος 2k+1, διαχωρίζουμε τα άρτια από τα περιττά a_m και, προσθέτοντάς τα ανά δύο, κατασκευάζουμε k ζεύγη όρων, ξένα μεταξύ τους, όπου το κάθε ζεύγος έχει άρτιο άθροισμα. Έτσι, θεωρώντας τις ισοτιμίες \mod k των ημιαθροισμάτων σε κάθε ζεύγος, το πρόβλημα ανάγεται στην περίπτωση n = k χωρίς τον περιορισμό του αρτίου πλήθους (αφού πρόκειται για ζεύγη όρων) η οποία, για πλήθος k, γνωρίζουμε ότι είναι επιλύσιμη. Άρα f(2k) = 2k+1.


Δημήτρης Σκουτέρης

Τα μαθηματικά είναι η μοναδική επιστήμη που θα μπορούσε κανείς να εξακολουθήσει να ασκεί αν κάποτε ξυπνούσε και το σύμπαν δεν υπήρχε πλέον.
Απάντηση

Επιστροφή σε “Συνδυαστική - Προχωρημένο Επίπεδο (Seniors)”

Μέλη σε σύνδεση

Μέλη σε αυτήν τη Δ. Συζήτηση: Δεν υπάρχουν εγγεγραμμένα μέλη και 4 επισκέπτες